Momento angular total del campo libre de Dirac en términos de operadores de creación y aniquilación

Cuando se realiza una cuantificación canónica para el campo de Dirac, normalmente se comienza expandiendo el campo de la siguiente forma:

ψ ( X ) = d 3 pag ( 2 π ) 3 2 mi pag s { a ( pag , s ) tu ( pag , s ) mi i pag X + b ( pag , s ) v ( pag , s ) mi i pag X } ,
dónde tu ( pag , s ) mi i pag X y v ( pag , s ) mi i pag X son soluciones de ondas planas de energía positiva y negativa a la ecuación de Dirac que también son espinores propios de helicidad con valor propio s . Al imponer una relación de anticonmutación de igual tiempo para el operador de campo y el momento canónico, se llega a las relaciones de anticonmutación fermiónicas satisfechas por a ( pag , s ) , b ( pag , s ) y sus conjugados hermitianos, donde adquieren significado físico como operadores de creación y aniquilación.

Después de introducir una sabia definición de lo que significa vacío, se puede escribir la expresión para el hamiltoniano, cantidad de movimiento y carga del campo en términos de estos operadores de creación y aniquilación. Por ejemplo, la expresión para el impulso dice

PAG m = d 3 pag s pag m [ a ( pag , s ) a ( pag , s ) + b ( pag , s ) b ( pag , s ) ] .

Sin embargo, cuando se trata del momento angular, los libros de texto solo parecen estar dispuestos a darlo en términos de ψ ( X ) , es decir,

METRO m v = d 3 X   ψ ( X ) [ i ( X m v X v m ) + 1 2 σ m v ] ψ ( X ) ,
dónde σ m v es el conjunto de matrices que controlan el comportamiento del espinor de Dirac bajo la transformación de Lorentz. Sé que la expresión anterior es un caso especial de una fórmula más general que proviene del teorema de Noether, y me pregunto si existe una expresión clara para METRO m v en términos de a ( pag , s ) , b ( pag , s ) y sus conjugados hermitianos. Traté de sustituir en la expansión de ψ ( X ) , pero no me llevó muy lejos. En particular, dado que hay un operador derivado intercalado entre los espinores tu y v , no pude aplicar relaciones ortogonales y no veo cómo proceder.

Entonces mi pregunta es: ¿Existe una expresión ordenada para el momento angular total? METRO m v del campo libre de Dirac ψ ( X ) en términos de operadores de creación y aniquilación a ( pag , s ) ,   a ( pag , s ) ,   b ( pag , s ) , b ( pag , s ) ? Si existe tal expresión, entonces, ¿cómo puedo llegar a ella? ¡Gracias por tu ayuda!

pd Hay una pregunta relacionada, pero no idéntica , en este sitio, y no parece haber obtenido una respuesta satisfactoria.


ACTUALIZACIÓN: para aquellos que puedan estar interesados, esto es lo que encontré después de la respuesta de @Numrok:
METRO m v = d 3 pag s , s { a ( pag , s ) tu ( pag , s ) 2 mi pag [ i ( pag v pag m pag m pag v ) + 1 2 σ m v ] a ( pag , s ) tu ( pag , s ) 2 mi pag + b ( pag , s ) v T ( pag , s ) 2 mi pag [ i ( pag m pag v pag v pag m ) 1 2 σ m v T ] b ( pag , s ) v ( pag , s ) 2 mi pag } .
Dónde / pag 0 debe tomarse como cero, y todos / pag i con i ser un índice espacial actuará sobre cualquier pag 0 = mi pag a la derecha de la misma. (estoy usando el + , , , firma). No estoy seguro de si esta es la forma más simple porque todavía me parece un poco complicado. Espero que la gente no dude en comentar si tiene alguna idea o información adicional.

Respuestas (1)

Sugerencias:

  1. Introduce la primera ecuación (expansión de ψ ( X ) ) en la integral del momento angular. Tenga cuidado de etiquetar el giro en los operadores de escalera ( a , b ) y vector base ( tu , v ).
  2. Aplicar las derivadas espaciales a los términos exponenciales.
  3. Utilice las relaciones de conmutación de los operadores de escalera para el campo de Dirac .
  4. Realice una de las integrales para deshacerse de las funciones delta resultantes.
  5. Use las relaciones de completitud y ortonormalidad de los vectores base, potencialmente la propia ecuación de Dirac.

Estos consejos deberían guiarlo a través de los pasos conceptuales, el resto es (con suerte) un simple ejercicio de cálculo.

¡Gracias! No me di cuenta de que uno puede usar las relaciones anticonmutación de los operadores de escalera antes de realizar cualquier integral. Además de esto, la desaparición de los términos que involucran a ambos a y b , o ambos a y b , no es trivial para mí y tuve que aplicar las propiedades de transformación de los espinores de Dirac para ver esto.
Mi resultado se publica al final de mi pregunta original.